LSAT and Law School Admissions Forum

Get expert LSAT preparation and law school admissions advice from PowerScore Test Preparation.

 Administrator
PowerScore Staff
  • PowerScore Staff
  • Posts: 8916
  • Joined: Feb 02, 2011
|
#61030
Please post your questions below!
 Bruin96
  • Posts: 33
  • Joined: Sep 04, 2019
|
#72119
Hello,

For this question I originally put E and I'm still not sure how the right answer is A.

I had the following set ups.


Temp #1
Spot 1: V
Spot 2: R
Spot 3: N
Spot 4: K
Spot 5: S/J
Spot 6: J/S

Temp #2
Spot 1: R
Spot 2: N
Spot 3: V
Spot 4: K
Spot 5: S/J
Spot 6: J/S

Temp #3
Spot 1: R
Spot 2: V
Spot 3: N
Spot 4: K
Spot 5: S/J
Spot 6: J/S

Please Help! I typically only miss -1/2 on games but for this prep test I missed 5.
 Claire Horan
PowerScore Staff
  • PowerScore Staff
  • Posts: 408
  • Joined: Apr 18, 2016
|
#72137
Hi Bruin96,

Your templates do not represent all of the solutions possible for this game. It would be a helpful exercise for you to go back and see what you missed.

I'm a little confused how you approached this problem because your (incomplete) list of templates suggests that K is ALWAYS auctioned 4th, but in that case ALL of the answer choices would be correct. I suspect you read the question stem backwards. It asks for the sufficient condition that necessitates K being auctioned fourth.

If J is auctioned 5th, S must be auctioned 6th because of the V-K-S and R-N sequencing, and the requirement that N be 2nd or 3rd. R must be 1st or 2nd, N must be secind or third, and V cannot be auctioned 4th because then it would be immediately before J, violating a rule. That leaves only K that can be 4th.

In terms of a conditional statement:
J5 :arrow: K4

I hope this helps. Happy studying!
 Bruin96
  • Posts: 33
  • Joined: Sep 04, 2019
|
#72168
You are absolutely right, I misread the question stem. I assumed that K was in the 4th and the answer choices would provide me with an answer that "Must be correct". When I went back and redid the game, I realized the mistake that I made. Thank you for the help! I was able to redo the game without missing any questions!
 j199393
  • Posts: 14
  • Joined: Jul 22, 2020
|
#78970
Could someone please let me know what the best way to attack this question is? How do we know which variables in the answer choices to test first in order to be the most efficient? I wasn't sure where to start and ended up wasting a lot of time testing out all of the answer choices.

Thank you!
User avatar
 KelseyWoods
PowerScore Staff
  • PowerScore Staff
  • Posts: 1079
  • Joined: Jun 26, 2013
|
#79805
Hi j199393!

Having templates for this game would help. You could do your templates based on whether N is 2 or 3:

Template #1 (N is 2):
R     N     V     K     S/J     J/S

Template #2 (N is 3 and J is before N):
R/J     J/R     N     V     K     S

Template #3 (N is 3 and V is before N):
R/V     V/R     N     ( K :longline: S,     J)

With these templates, you can see that K can only be 4th in templates #1 & #3. We can get rid of answer choice (B) right away because N is 3rd in template #2 but K is not 4th. Likewise, we can get rid of answer choice (C) because R could be 2nd in template #2 and it would not force K to be 4th. For answer choices (D) and (E), we look at template #3 since it's the only template in which V could be either 1st or 2nd: does V being 1st or 2nd have any impact on where K goes in template #3? Nope! So (D) and (E) are out. That leaves us with (A). If J was 5th in template #1, K would have to be 4th. If J was 5th in template #3, K would also have to be 4th. So (A) is correct!

Hope this helps!

Best,
Kelsey
 theamazingrace
  • Posts: 59
  • Joined: Oct 17, 2020
|
#80597
Hi, is it right to say that this is a could be true question? Since spaces 5 and 6 could be either S or J if K must be 4th?
User avatar
 KelseyWoods
PowerScore Staff
  • PowerScore Staff
  • Posts: 1079
  • Joined: Jun 26, 2013
|
#81255
Hi theamazingrace!

This is a Justify question. We are looking for an answer choice that, if true, would force the Kahlo painting to be fourth. That's similar to a Justify question in Logical Reasoning in which you are looking for an answer choice that would prove the conclusion of the argument to be true. We're not looking for an answer choice that could be true. We're assuming that all of the answer choices are true. We're looking for the answer choice that, if true, would force K into that 4th slot.

Hope this helps!

Best,
Kelsey

Get the most out of your LSAT Prep Plus subscription.

Analyze and track your performance with our Testing and Analytics Package.